Does the conclusion escape you? Has understanding the tone of the passage gotten you down? Get help here.
StratosM330
Students
 
Posts: 1
Joined: Tue Jul 14, 2020 3:12 pm
 

Collection of CR questions too vague for LSAT

by StratosM330 Fri Jul 17, 2020 9:38 am

[size=100]As the title reveals, I have been preparing for LSAT for quite some time, but recently decided to apply for Business Schools and switched to GMAT.

Meanwhile I have gotten used to the rigor of LSAT Logical Reasoning questions. The last couple days I went through all GMAT Critical Reasoning questions from Kaplan's GMAT 800, Manhattanprep's AllGMAT Verbal book and Manhattanprep's Question Bank. For many of the questions I got wrong, I am still convinced that more than 1 of the answers could equally be treated as correct (even if the stem says "most reasonable", "most strengthens/weakens" etc.), and I am convinced that they would definitely not have been precise enough to be set for LSAT.

Therefore, I thought to initiate a thread where we can post GMAT CR questions that are too unprecise. I'll make the start:

Manhattanprep Verbal book:

Chapter 22, Pset1:

Premise 1: Museum A display --> undamaged + proven to be authentic
Premise 2: Doubts about Mycenaean vase's authenticity, which is currently on display
Premise 3: prove vase's authenticity --> pulverize + spectroscopic analysis
Conclusion: ?

In my opinion, both B and C are true!

B: If the museum were sufficiently diligent in proving the authenticity of the vase before displaying it, then it means they would have pulverized and sent it to spectroscopy (see premise 3). If they had done that, they would not have been able to display it (since it has suffered from damage). But they are displaying it (see premise 2), therefore they were definitely NOT sufficiently diligent.

C: Indeed, it will no longer display it, based on the same logic I wrote for answer B.

The only reason which comes to my mind of eliminating B is because the stimulus says "will display", indicating that the statements might be not valid for the moment, but for the future only. This is really too vague though, on LSAT it would have been very clearly indicated that it refers to future only if so.

Manhattanprep Question Bank:

Q19:

Premise: TV is becoming too expensive for typical consumer
Conclusion: state that TV/household > persons/household will be reversed

In my opinion, both A and C strengthen equally (or at least I can't see any difference why one strengthens more than the other)!

A: this trend could definitely be due to the fact that consumers begin not being able to afford a TV. Potential objections:

- a rise of 300% within one year is quite big, it might be due to another reason, or an outlier
- the rise might be quite high, but it can be the case that the percentage of the population conducting parties for those purposes is so small that even a 300% increase can't be strong enough to reverse the trend in the conclusion

C: Indeed, this can also be due to the expensiveness of TV. Potential objections:

- TV can also be used as a secondary source of information and entertainment
- how much is the increase in purchases of the other devices? 0.1%, 1%, 10%, 50%?

Neither answer is completely bullet-proof (they don't have to, as stem says "most" strongly supports), but I can't really say that one answer is more bullet-proof (= requires more assumptions) than the other.

Kaplan GMAT 800:

This book is generally not very rigorous. Quite many questions were vague here and would certainly not make it through LSAT. I'll only write those down where I definitely think more than 1 answers are correct.

Q7:

Frankly, I've never seen answer choices so vaguely related to the stimulus in the LSAT. If we think very rigorously about it, none of them (except maybe E) really needs to be assumed. But if we compare the choices with each other, let's compromise a bit in terms of rigor and focus on (B) and (C) only:

For me, if we agree that C does not have to be assumed, then this is also true for B! I agree with the explanation given for why C is not a necessary assumption, but we can apply the same logic to B: the argument is only talking about candidates who are committed in their political party! The assumption has to be that Adele Richardson is likely to support the policy decision made by the national leadership of her minor party, but not the candidate of ANY party! Maybe candidates of other parties do not support policies of their own party at all...

Q5 also deserves an honourable mention here, but rather because it is so weirdly and vaguely formulated that it is not easy to follow the logic intended by the writer of the argument.[/size=100]
Sage Pearce-Higgins
Forum Guests
 
Posts: 1336
Joined: Thu Apr 03, 2014 4:04 am
 

Re: Collection of CR questions too vague for LSAT

by Sage Pearce-Higgins Wed Jul 22, 2020 4:45 am

Thanks for taking time to post. I agree that this is an interesting topic, and a frustrating one too. I can say from a personal perspective that I found CR infuriating when I starting studying GMAT, perhaps for the same reasons as you. I found the language to be vague and sometimes ambiguous, especially when compared to the Philosophy texts I was used to studying (and writing) at university. However, with experience I've been convinced by GMAT Critical Reasoning, and think that the more "real world" expression of scenarios is a useful training for students. I don't know LSAT, so I'm afraid I can't compare, although I would imagine that the lawyers require a stricter (and perhaps more artificial) use of language than business people. As for the materials, it's a simple fact of economics that test-prep companies can't invest as much into developing their materials as GMAC does for its official resources. GMAC has a very stringent editing process and really considers every nuance in a way that we're not able to do. That said, I very rarely see Manhattan Prep problems that I consider to have issues. Let me take those problems one by one (however, please quote the full problem in the future):

All the Verbal Chapter 22 Problem Set

Premise 1: Museum A display --> undamaged + proven to be authentic
Premise 2: Doubts about Mycenaean vase's authenticity, which is currently on display
Premise 3: prove vase's authenticity --> pulverize + spectroscopic analysis
Conclusion: ?

In my opinion, both B and C are true!

B: If the museum were sufficiently diligent in proving the authenticity of the vase before displaying it, then it means they would have pulverized and sent it to spectroscopy (see premise 3). If they had done that, they would not have been able to display it (since it has suffered from damage). But they are displaying it (see premise 2), therefore they were definitely NOT sufficiently diligent.

C: Indeed, it will no longer display it, based on the same logic I wrote for answer B.

The only reason which comes to my mind of eliminating B is because the stimulus says "will display", indicating that the statements might be not valid for the moment, but for the future only. This is really too vague though, on LSAT it would have been very clearly indicated that it refers to future only if so.


I would absolutely defend this problem against a charge of imprecision. The argument (including the correct answer C) follows a famous pattern: If A, then B; If not-A, then B; therefore B. If the museum doesn't verify the authenticity of the object, then it won't display it; if the museum does verify the authenticity of the object, then it won't display it. Part of getting good at Critical Reasoning is honing your ability to see such logical patterns.

Answer B contains another trap that's worth being familiar with. The phrase 'sufficiently diligent' is a value judgement, i.e. it's a subjective opinion. There is no information given in the passage to support this opinion. What constitutes 'sufficiently diligent'? According to whom? It's quite possible that the museum did a very thorough job of checking the origins of the vase, but in this case made a rare mistake.

Question Bank Q19

For the first time in history, more televisions than people can be found in American households. According to recent research, the average household has 2.55 residents and contains 2.73 televisions. However, by employing such costly manufacturing processes as plasma technology and flat screens, televisions are becoming too expensive for the typical consumer. As a result, the average number of residents per household will again surpass the number of televisions.

Which of the following, if true, most strongly supports the argument above?

A House parties at which numerous individuals gather to view popular television shows on one television set have increased three hundred percent during the past year.
B More than one million legal immigrants enter the United States each year.
C New devices such as video-enabled personal digital assistants and music players are increasingly purchased for use as a primary source of information and entertainment.
D As new technologies become more commonplace, manufacturing and retail costs normally decline.
E As a result of technological advances, new televisions are increasingly enabled with some features, such as Internet browsing, traditionally associated with other household devices.

Premise: TV is becoming too expensive for typical consumer
Conclusion: state that TV/household > persons/household will be reversed

In my opinion, both A and C strengthen equally (or at least I can't see any difference why one strengthens more than the other)!

A: this trend could definitely be due to the fact that consumers begin not being able to afford a TV. Potential objections:

- a rise of 300% within one year is quite big, it might be due to another reason, or an outlier
- the rise might be quite high, but it can be the case that the percentage of the population conducting parties for those purposes is so small that even a 300% increase can't be strong enough to reverse the trend in the conclusion

C: Indeed, this can also be due to the expensiveness of TV. Potential objections:

- TV can also be used as a secondary source of information and entertainment
- how much is the increase in purchases of the other devices? 0.1%, 1%, 10%, 50%?

Neither answer is completely bullet-proof (they don't have to, as stem says "most" strongly supports), but I can't really say that one answer is more bullet-proof (= requires more assumptions) than the other.


I would also defend this problem against the charge of imprecision. I agree with you that 'neither answer is completely bullet-proof', but that's not actually the point. This is real-world reasoning, and words are, of course, less precise than numbers. These problems are based on practical situations in which there is always a good deal of reasonable doubt, and the correct answer in strengthen / weaken problems is not expected to produce 100% certainty. Step back from the problem and think of it in common sense, everyday terms. Which of these two factors would have the biggest influence on whether Americans buy TVs: house parties, or people watching movies on their phones? Clearly the second. Answer C contains a pattern worth recognizing: this answer is giving another reason why Americans might not want to buy TVs.

As for the Kaplan problem, I don't have access to that book, but feel free to post the problem in the General Verbal forum and I could comment on it.